Search found 199 matches


Hi guys, does the GMAT ask questions on base system? For example: how would you write 2234 in base 4 to a number in base 7? 2234 in base 4 (2 x 4^3) + (2 x 4^2) + (2 x 4) + 4 = 128 + 32 + 8 + 4 = 172 (Base 10) In Base 7... Wasn't sure how to do this formulaically, so I just logically reasoned it ou...

by 800_or_bust

Tue Jul 12, 2016 6:10 am
Forum: GMAT Math
Topic: Questions on base system
Replies: 14
Views: 7226

Man, you've nailed it! Congratulations! Wish I had your brain while taking the GMAT! :) Marty, what's up with being vegan and scoring high on verbal? What's the link?! :D Also, this Chinese thing, it seems like Suits to me. Nuts and messed up! Congrats once again 800_or_bust! Thanks Diego! I've rea...

by 800_or_bust

Sun Jul 10, 2016 6:42 am
Forum: GMAT Strategy
Topic: My Exam Preparation Log (formerly Just Starting Out...)
Replies: 56
Views: 13769

So I completed my official GMAT this morning. Finished with a 780 (51Q/47V/8IR). The proctor stated that tied the highest score she had ever seen on the GMAT. Nice work. But I was a little disappointed I wasn't able to get over the 800 hump, especially since my last practice exam was an 800 (51Q/51...

by 800_or_bust

Sun Jul 10, 2016 6:25 am
Forum: GMAT Strategy
Topic: My Exam Preparation Log (formerly Just Starting Out...)
Replies: 56
Views: 13769

Remember for all a & b, a - b = - (b - a). And (a-b)^2 = (b-a)^2, because squaring always produces a positive result and each of those expressions have the same absolute value.

by 800_or_bust

Sun Jul 10, 2016 6:09 am
Forum: Problem Solving
Topic: Quadratic Equation - Why is this the answer?
Replies: 6
Views: 1701

The correct way to approach this algebraically would be to factor out a -1 from the denominator of the second term as follows: ((a^2 + 12) / (a-3)) + (7a / (3-a)) = ((a^2 + 12) / (a-3)) + (7a / ((-1)(a-3)) Now the -1 can be freely moved to the numerator, so it simplifies as follows: ((a^2 + 12) / (a...

by 800_or_bust

Sun Jul 10, 2016 6:03 am
Forum: Problem Solving
Topic: Quadratic Equation - Why is this the answer?
Replies: 6
Views: 1701

Attached is a copy of my work. The answer is B but I got D and E using the shortcut for getting common denominators. Any help would be greatly appreciated. You were good up to the point where you combined the fraction, but then you cancelled terms that couldn't be cancelled. And where did the 0 com...

by 800_or_bust

Sun Jul 10, 2016 5:49 am
Forum: Problem Solving
Topic: Quadratic Equation - Why is this the answer?
Replies: 6
Views: 1701

So I completed my official GMAT this morning. Finished with a 780 (51Q/47V/8IR). The proctor stated that tied the highest score she had ever seen on the GMAT. But I was a little disappointed I wasn't able to get over the 800 hump, especially since my last practice exam was an 800 (51Q/51V/8IR). It's...

by 800_or_bust

Sat Jul 09, 2016 8:07 pm
Forum: GMAT Strategy
Topic: My Exam Preparation Log (formerly Just Starting Out...)
Replies: 56
Views: 13769

Max@Math Revolution wrote:Is a=b?
1) a^2=b^2
2) a=1

*An answer will be posted in 2 days
(1) NOT SUFFICIENT. A could equal B, or A could equal -B.
(2) NOT SUFFICIENT. B could be any value.

Combined (1) & (2). NOT SUFFICIENT. A = 1, B could equal 1 or -1.

by 800_or_bust

Sat Jul 09, 2016 5:14 am
Forum: Data Sufficiency
Topic: Is a=b?
Replies: 3
Views: 1336

If xy > 0, is (xy)^2 < √(xy) (1) 4/x > 7y (2) x − 16 > −16 Note that the inequality in the question prompt will be true for values of xy, such that 0 < xy < 1. So the question can be rephrased as "Is 0<xy<1?" (1) NOT SUFFICIENT. If x > 0, the inequality becomes 4/7 > xy. Which combi...

by 800_or_bust

Fri Jul 08, 2016 5:38 pm
Forum: Data Sufficiency
Topic: Inequality
Replies: 3
Views: 1290

Another example of this question type from the Official Guide: http://gmatclub.com/forum/is-the-positi ... 67953.html

by 800_or_bust

Fri Jul 08, 2016 2:48 pm
Forum: Data Sufficiency
Topic: Data sufficiency
Replies: 5
Views: 1702

Q. 12 children take 16 days to complete the work which can be completed by 8 adults in 12 days.16 adults started work and after 3 days 10 adults left & 4 children join them.How many days will they take to complete the work. A.3; B.4; C.6; D.8; E.None Assuming the children and adults all work at...

by 800_or_bust

Fri Jul 08, 2016 2:01 pm
Forum: Problem Solving
Topic: Time & Work
Replies: 4
Views: 6454

If n is a positive integer, is (n^3 – n) divisible by 4 ? (1) n = 2k + 1, where k is an integer. (2) n2 + n is divisible by 6. This is a q from OG12 and the answer is A. My question is why can't we consider the value of K as 0 which would make even the first condition NS. Please help. Thank you.....

by 800_or_bust

Fri Jul 08, 2016 9:35 am
Forum: Data Sufficiency
Topic: If we consider the value of K as 0
Replies: 8
Views: 2347

Sets M and N contain exactly m and n elements, respectively. What is the value of n? (1) 7m=8n (2) The intersection of M and N contains exactly 0.4m elements. (1) NOT SUFFICIENT. We know m & n must be positive integers, but there are numerous pairs for which this equation holds. m = 8, n = 7; m...

by 800_or_bust

Thu Jul 07, 2016 11:23 am
Forum: Problem Solving
Topic: Combined sets and ratio problem
Replies: 3
Views: 4842

Joy Shaha wrote:Q. -5<s<5,, is s> o?
S:S>4
1/s > s/1
I think you transcribed something incorrectly. What does (1) even mean - the ratio of S to S equals 4?!? Also should be in the DS forum.

by 800_or_bust

Thu Jul 07, 2016 10:55 am
Forum: Problem Solving
Topic: Data sufficiency
Replies: 2
Views: 940

https://s31.postimg.org/q84k8g5pj/Cistern.jpg The 4 taps can fill the entire 500L tank in 12.5, and they are identical. Hence, each tap fills 125L every 12.5 hours, or 10L every one hour, or 5L every 30 minutes. The 4 taps together would fill 20L every 30 minutes. The 5 emptying taps can completely...

by 800_or_bust

Thu Jul 07, 2016 10:47 am
Forum: Problem Solving
Topic: Time related
Replies: 2
Views: 1105